Evaluate $int_0^1 log left( frac{x^2+sqrt{3}x+1}{x^2-sqrt{3}x+1} right) frac{dx}{x} $












5












$begingroup$


I'm trying to evaluate this integral but i'm having a lot of problems with the standards method.



$$int_0^1
log
left(
frac{x^2+sqrt{3}x+1}{x^2-sqrt{3}x+1}
right)
frac{dx}{x}
$$



I've tried integration by parts, taylor expansion of the $log$ function and some substitution.



Wolfram says that the results is $frac{pi^2}{3}$. I think that it divide the integral in four parts factorizing the argument of the logarithm with complex numbers, anyway it seems to take too long as approach.
Do you have any suggestions?



I've also noticed that the function is even so we could also write the integral as
$$
frac{1}{2}
int_{-1}^1
log
left(
frac{x^2+sqrt{3}x+1}{x^2-sqrt{3}x+1}
right)
frac{dx}{x}
$$










share|cite|improve this question









$endgroup$








  • 1




    $begingroup$
    Both the numerator and denominator are invariant under the transformation $p(x) mapsto x^2 p(frac{1}{x})$, which suggests you might get some mileage out of applying the substitution $u = frac{1}{x}$ and rearranging.
    $endgroup$
    – Travis
    Jan 7 at 13:13
















5












$begingroup$


I'm trying to evaluate this integral but i'm having a lot of problems with the standards method.



$$int_0^1
log
left(
frac{x^2+sqrt{3}x+1}{x^2-sqrt{3}x+1}
right)
frac{dx}{x}
$$



I've tried integration by parts, taylor expansion of the $log$ function and some substitution.



Wolfram says that the results is $frac{pi^2}{3}$. I think that it divide the integral in four parts factorizing the argument of the logarithm with complex numbers, anyway it seems to take too long as approach.
Do you have any suggestions?



I've also noticed that the function is even so we could also write the integral as
$$
frac{1}{2}
int_{-1}^1
log
left(
frac{x^2+sqrt{3}x+1}{x^2-sqrt{3}x+1}
right)
frac{dx}{x}
$$










share|cite|improve this question









$endgroup$








  • 1




    $begingroup$
    Both the numerator and denominator are invariant under the transformation $p(x) mapsto x^2 p(frac{1}{x})$, which suggests you might get some mileage out of applying the substitution $u = frac{1}{x}$ and rearranging.
    $endgroup$
    – Travis
    Jan 7 at 13:13














5












5








5





$begingroup$


I'm trying to evaluate this integral but i'm having a lot of problems with the standards method.



$$int_0^1
log
left(
frac{x^2+sqrt{3}x+1}{x^2-sqrt{3}x+1}
right)
frac{dx}{x}
$$



I've tried integration by parts, taylor expansion of the $log$ function and some substitution.



Wolfram says that the results is $frac{pi^2}{3}$. I think that it divide the integral in four parts factorizing the argument of the logarithm with complex numbers, anyway it seems to take too long as approach.
Do you have any suggestions?



I've also noticed that the function is even so we could also write the integral as
$$
frac{1}{2}
int_{-1}^1
log
left(
frac{x^2+sqrt{3}x+1}{x^2-sqrt{3}x+1}
right)
frac{dx}{x}
$$










share|cite|improve this question









$endgroup$




I'm trying to evaluate this integral but i'm having a lot of problems with the standards method.



$$int_0^1
log
left(
frac{x^2+sqrt{3}x+1}{x^2-sqrt{3}x+1}
right)
frac{dx}{x}
$$



I've tried integration by parts, taylor expansion of the $log$ function and some substitution.



Wolfram says that the results is $frac{pi^2}{3}$. I think that it divide the integral in four parts factorizing the argument of the logarithm with complex numbers, anyway it seems to take too long as approach.
Do you have any suggestions?



I've also noticed that the function is even so we could also write the integral as
$$
frac{1}{2}
int_{-1}^1
log
left(
frac{x^2+sqrt{3}x+1}{x^2-sqrt{3}x+1}
right)
frac{dx}{x}
$$







definite-integrals






share|cite|improve this question













share|cite|improve this question











share|cite|improve this question




share|cite|improve this question










asked Jan 7 at 13:05









FabioFabio

1029




1029








  • 1




    $begingroup$
    Both the numerator and denominator are invariant under the transformation $p(x) mapsto x^2 p(frac{1}{x})$, which suggests you might get some mileage out of applying the substitution $u = frac{1}{x}$ and rearranging.
    $endgroup$
    – Travis
    Jan 7 at 13:13














  • 1




    $begingroup$
    Both the numerator and denominator are invariant under the transformation $p(x) mapsto x^2 p(frac{1}{x})$, which suggests you might get some mileage out of applying the substitution $u = frac{1}{x}$ and rearranging.
    $endgroup$
    – Travis
    Jan 7 at 13:13








1




1




$begingroup$
Both the numerator and denominator are invariant under the transformation $p(x) mapsto x^2 p(frac{1}{x})$, which suggests you might get some mileage out of applying the substitution $u = frac{1}{x}$ and rearranging.
$endgroup$
– Travis
Jan 7 at 13:13




$begingroup$
Both the numerator and denominator are invariant under the transformation $p(x) mapsto x^2 p(frac{1}{x})$, which suggests you might get some mileage out of applying the substitution $u = frac{1}{x}$ and rearranging.
$endgroup$
– Travis
Jan 7 at 13:13










2 Answers
2






active

oldest

votes


















13












$begingroup$

Let $$I(theta)=int_0^1frac{ln(x^2+2xcostheta+1)}xdx,$$
differentiating it gives $$I'(theta)=int_0^1-frac{2sintheta}{1+2xcostheta+x^2}dx\
=-theta$$

(since $thetain(-pi,pi)$)

The integral need to find is $$Ileft(frac16piright)-Ileft(frac56piright)=int_{5/6pi}^{1/6pi}-theta dtheta=frac13pi^2$$






share|cite|improve this answer









$endgroup$













  • $begingroup$
    Brilliant, i'll add this trick to my collection thank you
    $endgroup$
    – Fabio
    Jan 7 at 13:24






  • 1




    $begingroup$
    @Fabio one can actually generalize this method to see that $$int_0^1logbigg(frac{x^2+ax+1}{x^2-ax+1}bigg)frac{mathrm{d}x}{x}=frac{pi^2}2-piarccosfrac{a}2$$
    $endgroup$
    – clathratus
    2 days ago










  • $begingroup$
    I didn't noticed, besides allow to give a more elegant answer I guess this will be quite useful in the future, thanks for the observation
    $endgroup$
    – Fabio
    2 days ago



















8












$begingroup$

Let us consider that for any $alphain S^1$



$$ int_{0}^{1}frac{log(1-alpha x)}{x},dx = -sum_{ngeq 1}int_{0}^{1}frac{alpha^n x^{n}}{nx},dx=-sum_{ngeq 1}frac{alpha^n}{n^2}=-text{Li}_2(alpha) $$
and by setting $zeta=expleft(frac{2pi i}{12}right)$ we have



$$ int_{0}^{1}logleft(frac{1-sqrt{3}x+x^2}{1+sqrt{3}x+x^2}right),frac{dx}{x}=int_{0}^{1}left[log(1-zeta x)+log(1-zeta^{11} x)-log(1-zeta^5 x)-log(1-zeta^7 x)right]frac{dx}{x} $$
hence your integral equals
$$ text{Li}_2(zeta)+text{Li}_2(zeta^{11})-text{Li}_2(zeta^5)-text{Li}_2(zeta^7)=2text{Re},text{Li}_2(zeta)-2text{Re},text{Li}_2(zeta^5). $$
Last trick: $text{Re},text{Li}_2(e^{itheta})$ is an elementary function, since it is the formal primitive of the sawtooth wave $sum_{ngeq 1}frac{sin(nx)}{n}$ (piecewise-linear), hence a periodic and piecewise-parabolic function. Putting everything together, the outcome is simply $2zeta(2)=frac{pi^2}{3}$.






share|cite|improve this answer











$endgroup$













  • $begingroup$
    That's impressive! But i don't understand some things sorry for the ignorance: what's the set $S^1$?
    $endgroup$
    – Fabio
    Jan 7 at 13:31






  • 1




    $begingroup$
    @Fabio: the unit circle centered at the origin of the complex plane.
    $endgroup$
    – Jack D'Aurizio
    Jan 7 at 13:52











Your Answer





StackExchange.ifUsing("editor", function () {
return StackExchange.using("mathjaxEditing", function () {
StackExchange.MarkdownEditor.creationCallbacks.add(function (editor, postfix) {
StackExchange.mathjaxEditing.prepareWmdForMathJax(editor, postfix, [["$", "$"], ["\\(","\\)"]]);
});
});
}, "mathjax-editing");

StackExchange.ready(function() {
var channelOptions = {
tags: "".split(" "),
id: "69"
};
initTagRenderer("".split(" "), "".split(" "), channelOptions);

StackExchange.using("externalEditor", function() {
// Have to fire editor after snippets, if snippets enabled
if (StackExchange.settings.snippets.snippetsEnabled) {
StackExchange.using("snippets", function() {
createEditor();
});
}
else {
createEditor();
}
});

function createEditor() {
StackExchange.prepareEditor({
heartbeatType: 'answer',
autoActivateHeartbeat: false,
convertImagesToLinks: true,
noModals: true,
showLowRepImageUploadWarning: true,
reputationToPostImages: 10,
bindNavPrevention: true,
postfix: "",
imageUploader: {
brandingHtml: "Powered by u003ca class="icon-imgur-white" href="https://imgur.com/"u003eu003c/au003e",
contentPolicyHtml: "User contributions licensed under u003ca href="https://creativecommons.org/licenses/by-sa/3.0/"u003ecc by-sa 3.0 with attribution requiredu003c/au003e u003ca href="https://stackoverflow.com/legal/content-policy"u003e(content policy)u003c/au003e",
allowUrls: true
},
noCode: true, onDemand: true,
discardSelector: ".discard-answer"
,immediatelyShowMarkdownHelp:true
});


}
});














draft saved

draft discarded


















StackExchange.ready(
function () {
StackExchange.openid.initPostLogin('.new-post-login', 'https%3a%2f%2fmath.stackexchange.com%2fquestions%2f3064983%2fevaluate-int-01-log-left-fracx2-sqrt3x1x2-sqrt3x1-right%23new-answer', 'question_page');
}
);

Post as a guest















Required, but never shown

























2 Answers
2






active

oldest

votes








2 Answers
2






active

oldest

votes









active

oldest

votes






active

oldest

votes









13












$begingroup$

Let $$I(theta)=int_0^1frac{ln(x^2+2xcostheta+1)}xdx,$$
differentiating it gives $$I'(theta)=int_0^1-frac{2sintheta}{1+2xcostheta+x^2}dx\
=-theta$$

(since $thetain(-pi,pi)$)

The integral need to find is $$Ileft(frac16piright)-Ileft(frac56piright)=int_{5/6pi}^{1/6pi}-theta dtheta=frac13pi^2$$






share|cite|improve this answer









$endgroup$













  • $begingroup$
    Brilliant, i'll add this trick to my collection thank you
    $endgroup$
    – Fabio
    Jan 7 at 13:24






  • 1




    $begingroup$
    @Fabio one can actually generalize this method to see that $$int_0^1logbigg(frac{x^2+ax+1}{x^2-ax+1}bigg)frac{mathrm{d}x}{x}=frac{pi^2}2-piarccosfrac{a}2$$
    $endgroup$
    – clathratus
    2 days ago










  • $begingroup$
    I didn't noticed, besides allow to give a more elegant answer I guess this will be quite useful in the future, thanks for the observation
    $endgroup$
    – Fabio
    2 days ago
















13












$begingroup$

Let $$I(theta)=int_0^1frac{ln(x^2+2xcostheta+1)}xdx,$$
differentiating it gives $$I'(theta)=int_0^1-frac{2sintheta}{1+2xcostheta+x^2}dx\
=-theta$$

(since $thetain(-pi,pi)$)

The integral need to find is $$Ileft(frac16piright)-Ileft(frac56piright)=int_{5/6pi}^{1/6pi}-theta dtheta=frac13pi^2$$






share|cite|improve this answer









$endgroup$













  • $begingroup$
    Brilliant, i'll add this trick to my collection thank you
    $endgroup$
    – Fabio
    Jan 7 at 13:24






  • 1




    $begingroup$
    @Fabio one can actually generalize this method to see that $$int_0^1logbigg(frac{x^2+ax+1}{x^2-ax+1}bigg)frac{mathrm{d}x}{x}=frac{pi^2}2-piarccosfrac{a}2$$
    $endgroup$
    – clathratus
    2 days ago










  • $begingroup$
    I didn't noticed, besides allow to give a more elegant answer I guess this will be quite useful in the future, thanks for the observation
    $endgroup$
    – Fabio
    2 days ago














13












13








13





$begingroup$

Let $$I(theta)=int_0^1frac{ln(x^2+2xcostheta+1)}xdx,$$
differentiating it gives $$I'(theta)=int_0^1-frac{2sintheta}{1+2xcostheta+x^2}dx\
=-theta$$

(since $thetain(-pi,pi)$)

The integral need to find is $$Ileft(frac16piright)-Ileft(frac56piright)=int_{5/6pi}^{1/6pi}-theta dtheta=frac13pi^2$$






share|cite|improve this answer









$endgroup$



Let $$I(theta)=int_0^1frac{ln(x^2+2xcostheta+1)}xdx,$$
differentiating it gives $$I'(theta)=int_0^1-frac{2sintheta}{1+2xcostheta+x^2}dx\
=-theta$$

(since $thetain(-pi,pi)$)

The integral need to find is $$Ileft(frac16piright)-Ileft(frac56piright)=int_{5/6pi}^{1/6pi}-theta dtheta=frac13pi^2$$







share|cite|improve this answer












share|cite|improve this answer



share|cite|improve this answer










answered Jan 7 at 13:21









Kemono ChenKemono Chen

2,8531738




2,8531738












  • $begingroup$
    Brilliant, i'll add this trick to my collection thank you
    $endgroup$
    – Fabio
    Jan 7 at 13:24






  • 1




    $begingroup$
    @Fabio one can actually generalize this method to see that $$int_0^1logbigg(frac{x^2+ax+1}{x^2-ax+1}bigg)frac{mathrm{d}x}{x}=frac{pi^2}2-piarccosfrac{a}2$$
    $endgroup$
    – clathratus
    2 days ago










  • $begingroup$
    I didn't noticed, besides allow to give a more elegant answer I guess this will be quite useful in the future, thanks for the observation
    $endgroup$
    – Fabio
    2 days ago


















  • $begingroup$
    Brilliant, i'll add this trick to my collection thank you
    $endgroup$
    – Fabio
    Jan 7 at 13:24






  • 1




    $begingroup$
    @Fabio one can actually generalize this method to see that $$int_0^1logbigg(frac{x^2+ax+1}{x^2-ax+1}bigg)frac{mathrm{d}x}{x}=frac{pi^2}2-piarccosfrac{a}2$$
    $endgroup$
    – clathratus
    2 days ago










  • $begingroup$
    I didn't noticed, besides allow to give a more elegant answer I guess this will be quite useful in the future, thanks for the observation
    $endgroup$
    – Fabio
    2 days ago
















$begingroup$
Brilliant, i'll add this trick to my collection thank you
$endgroup$
– Fabio
Jan 7 at 13:24




$begingroup$
Brilliant, i'll add this trick to my collection thank you
$endgroup$
– Fabio
Jan 7 at 13:24




1




1




$begingroup$
@Fabio one can actually generalize this method to see that $$int_0^1logbigg(frac{x^2+ax+1}{x^2-ax+1}bigg)frac{mathrm{d}x}{x}=frac{pi^2}2-piarccosfrac{a}2$$
$endgroup$
– clathratus
2 days ago




$begingroup$
@Fabio one can actually generalize this method to see that $$int_0^1logbigg(frac{x^2+ax+1}{x^2-ax+1}bigg)frac{mathrm{d}x}{x}=frac{pi^2}2-piarccosfrac{a}2$$
$endgroup$
– clathratus
2 days ago












$begingroup$
I didn't noticed, besides allow to give a more elegant answer I guess this will be quite useful in the future, thanks for the observation
$endgroup$
– Fabio
2 days ago




$begingroup$
I didn't noticed, besides allow to give a more elegant answer I guess this will be quite useful in the future, thanks for the observation
$endgroup$
– Fabio
2 days ago











8












$begingroup$

Let us consider that for any $alphain S^1$



$$ int_{0}^{1}frac{log(1-alpha x)}{x},dx = -sum_{ngeq 1}int_{0}^{1}frac{alpha^n x^{n}}{nx},dx=-sum_{ngeq 1}frac{alpha^n}{n^2}=-text{Li}_2(alpha) $$
and by setting $zeta=expleft(frac{2pi i}{12}right)$ we have



$$ int_{0}^{1}logleft(frac{1-sqrt{3}x+x^2}{1+sqrt{3}x+x^2}right),frac{dx}{x}=int_{0}^{1}left[log(1-zeta x)+log(1-zeta^{11} x)-log(1-zeta^5 x)-log(1-zeta^7 x)right]frac{dx}{x} $$
hence your integral equals
$$ text{Li}_2(zeta)+text{Li}_2(zeta^{11})-text{Li}_2(zeta^5)-text{Li}_2(zeta^7)=2text{Re},text{Li}_2(zeta)-2text{Re},text{Li}_2(zeta^5). $$
Last trick: $text{Re},text{Li}_2(e^{itheta})$ is an elementary function, since it is the formal primitive of the sawtooth wave $sum_{ngeq 1}frac{sin(nx)}{n}$ (piecewise-linear), hence a periodic and piecewise-parabolic function. Putting everything together, the outcome is simply $2zeta(2)=frac{pi^2}{3}$.






share|cite|improve this answer











$endgroup$













  • $begingroup$
    That's impressive! But i don't understand some things sorry for the ignorance: what's the set $S^1$?
    $endgroup$
    – Fabio
    Jan 7 at 13:31






  • 1




    $begingroup$
    @Fabio: the unit circle centered at the origin of the complex plane.
    $endgroup$
    – Jack D'Aurizio
    Jan 7 at 13:52
















8












$begingroup$

Let us consider that for any $alphain S^1$



$$ int_{0}^{1}frac{log(1-alpha x)}{x},dx = -sum_{ngeq 1}int_{0}^{1}frac{alpha^n x^{n}}{nx},dx=-sum_{ngeq 1}frac{alpha^n}{n^2}=-text{Li}_2(alpha) $$
and by setting $zeta=expleft(frac{2pi i}{12}right)$ we have



$$ int_{0}^{1}logleft(frac{1-sqrt{3}x+x^2}{1+sqrt{3}x+x^2}right),frac{dx}{x}=int_{0}^{1}left[log(1-zeta x)+log(1-zeta^{11} x)-log(1-zeta^5 x)-log(1-zeta^7 x)right]frac{dx}{x} $$
hence your integral equals
$$ text{Li}_2(zeta)+text{Li}_2(zeta^{11})-text{Li}_2(zeta^5)-text{Li}_2(zeta^7)=2text{Re},text{Li}_2(zeta)-2text{Re},text{Li}_2(zeta^5). $$
Last trick: $text{Re},text{Li}_2(e^{itheta})$ is an elementary function, since it is the formal primitive of the sawtooth wave $sum_{ngeq 1}frac{sin(nx)}{n}$ (piecewise-linear), hence a periodic and piecewise-parabolic function. Putting everything together, the outcome is simply $2zeta(2)=frac{pi^2}{3}$.






share|cite|improve this answer











$endgroup$













  • $begingroup$
    That's impressive! But i don't understand some things sorry for the ignorance: what's the set $S^1$?
    $endgroup$
    – Fabio
    Jan 7 at 13:31






  • 1




    $begingroup$
    @Fabio: the unit circle centered at the origin of the complex plane.
    $endgroup$
    – Jack D'Aurizio
    Jan 7 at 13:52














8












8








8





$begingroup$

Let us consider that for any $alphain S^1$



$$ int_{0}^{1}frac{log(1-alpha x)}{x},dx = -sum_{ngeq 1}int_{0}^{1}frac{alpha^n x^{n}}{nx},dx=-sum_{ngeq 1}frac{alpha^n}{n^2}=-text{Li}_2(alpha) $$
and by setting $zeta=expleft(frac{2pi i}{12}right)$ we have



$$ int_{0}^{1}logleft(frac{1-sqrt{3}x+x^2}{1+sqrt{3}x+x^2}right),frac{dx}{x}=int_{0}^{1}left[log(1-zeta x)+log(1-zeta^{11} x)-log(1-zeta^5 x)-log(1-zeta^7 x)right]frac{dx}{x} $$
hence your integral equals
$$ text{Li}_2(zeta)+text{Li}_2(zeta^{11})-text{Li}_2(zeta^5)-text{Li}_2(zeta^7)=2text{Re},text{Li}_2(zeta)-2text{Re},text{Li}_2(zeta^5). $$
Last trick: $text{Re},text{Li}_2(e^{itheta})$ is an elementary function, since it is the formal primitive of the sawtooth wave $sum_{ngeq 1}frac{sin(nx)}{n}$ (piecewise-linear), hence a periodic and piecewise-parabolic function. Putting everything together, the outcome is simply $2zeta(2)=frac{pi^2}{3}$.






share|cite|improve this answer











$endgroup$



Let us consider that for any $alphain S^1$



$$ int_{0}^{1}frac{log(1-alpha x)}{x},dx = -sum_{ngeq 1}int_{0}^{1}frac{alpha^n x^{n}}{nx},dx=-sum_{ngeq 1}frac{alpha^n}{n^2}=-text{Li}_2(alpha) $$
and by setting $zeta=expleft(frac{2pi i}{12}right)$ we have



$$ int_{0}^{1}logleft(frac{1-sqrt{3}x+x^2}{1+sqrt{3}x+x^2}right),frac{dx}{x}=int_{0}^{1}left[log(1-zeta x)+log(1-zeta^{11} x)-log(1-zeta^5 x)-log(1-zeta^7 x)right]frac{dx}{x} $$
hence your integral equals
$$ text{Li}_2(zeta)+text{Li}_2(zeta^{11})-text{Li}_2(zeta^5)-text{Li}_2(zeta^7)=2text{Re},text{Li}_2(zeta)-2text{Re},text{Li}_2(zeta^5). $$
Last trick: $text{Re},text{Li}_2(e^{itheta})$ is an elementary function, since it is the formal primitive of the sawtooth wave $sum_{ngeq 1}frac{sin(nx)}{n}$ (piecewise-linear), hence a periodic and piecewise-parabolic function. Putting everything together, the outcome is simply $2zeta(2)=frac{pi^2}{3}$.







share|cite|improve this answer














share|cite|improve this answer



share|cite|improve this answer








edited Jan 7 at 18:21

























answered Jan 7 at 13:25









Jack D'AurizioJack D'Aurizio

288k33280659




288k33280659












  • $begingroup$
    That's impressive! But i don't understand some things sorry for the ignorance: what's the set $S^1$?
    $endgroup$
    – Fabio
    Jan 7 at 13:31






  • 1




    $begingroup$
    @Fabio: the unit circle centered at the origin of the complex plane.
    $endgroup$
    – Jack D'Aurizio
    Jan 7 at 13:52


















  • $begingroup$
    That's impressive! But i don't understand some things sorry for the ignorance: what's the set $S^1$?
    $endgroup$
    – Fabio
    Jan 7 at 13:31






  • 1




    $begingroup$
    @Fabio: the unit circle centered at the origin of the complex plane.
    $endgroup$
    – Jack D'Aurizio
    Jan 7 at 13:52
















$begingroup$
That's impressive! But i don't understand some things sorry for the ignorance: what's the set $S^1$?
$endgroup$
– Fabio
Jan 7 at 13:31




$begingroup$
That's impressive! But i don't understand some things sorry for the ignorance: what's the set $S^1$?
$endgroup$
– Fabio
Jan 7 at 13:31




1




1




$begingroup$
@Fabio: the unit circle centered at the origin of the complex plane.
$endgroup$
– Jack D'Aurizio
Jan 7 at 13:52




$begingroup$
@Fabio: the unit circle centered at the origin of the complex plane.
$endgroup$
– Jack D'Aurizio
Jan 7 at 13:52


















draft saved

draft discarded




















































Thanks for contributing an answer to Mathematics Stack Exchange!


  • Please be sure to answer the question. Provide details and share your research!

But avoid



  • Asking for help, clarification, or responding to other answers.

  • Making statements based on opinion; back them up with references or personal experience.


Use MathJax to format equations. MathJax reference.


To learn more, see our tips on writing great answers.




draft saved


draft discarded














StackExchange.ready(
function () {
StackExchange.openid.initPostLogin('.new-post-login', 'https%3a%2f%2fmath.stackexchange.com%2fquestions%2f3064983%2fevaluate-int-01-log-left-fracx2-sqrt3x1x2-sqrt3x1-right%23new-answer', 'question_page');
}
);

Post as a guest















Required, but never shown





















































Required, but never shown














Required, but never shown












Required, but never shown







Required, but never shown

































Required, but never shown














Required, but never shown












Required, but never shown







Required, but never shown







Popular posts from this blog

Mario Kart Wii

What does “Dominus providebit” mean?

Antonio Litta Visconti Arese